Difference between revisions of "2018 AMC 10A Problems/Problem 7"
(→Video Solution) |
(→Solution) |
||
Line 9: | Line 9: | ||
\textbf{(E) }9 \qquad | \textbf{(E) }9 \qquad | ||
</math> | </math> | ||
+ | |||
+ | ==Video Solution== | ||
+ | https://youtu.be/vzyRAnpnJes | ||
+ | |||
+ | Education, the Study of Everything | ||
+ | |||
+ | |||
==Solution== | ==Solution== |
Revision as of 10:49, 29 November 2020
Problem
For how many (not necessarily positive) integer values of is the value of an integer?
Video Solution
Education, the Study of Everything
Solution
The prime factorization of is . Therefore, the maximum number for is , and the minimum number for is . Then we must find the range from to , which is .
Video Solution
~savannahsolver
See Also
2018 AMC 10A (Problems • Answer Key • Resources) | ||
Preceded by Problem 6 |
Followed by Problem 8 | |
1 • 2 • 3 • 4 • 5 • 6 • 7 • 8 • 9 • 10 • 11 • 12 • 13 • 14 • 15 • 16 • 17 • 18 • 19 • 20 • 21 • 22 • 23 • 24 • 25 | ||
All AMC 10 Problems and Solutions |
2018 AMC 12A (Problems • Answer Key • Resources) | |
Preceded by Problem 6 |
Followed by Problem 8 |
1 • 2 • 3 • 4 • 5 • 6 • 7 • 8 • 9 • 10 • 11 • 12 • 13 • 14 • 15 • 16 • 17 • 18 • 19 • 20 • 21 • 22 • 23 • 24 • 25 | |
All AMC 12 Problems and Solutions |
The problems on this page are copyrighted by the Mathematical Association of America's American Mathematics Competitions.